Difference between revisions of "Mock AIME 3 Pre 2005 Problems/Problem 4"

m
(solution)
Line 1: Line 1:
 
==Problem==
 
==Problem==
<math>\zeta_1, \zeta_2,</math> and <math>\zeta_3</math> are complex numbers such that
+
<math>\zeta_1, \zeta_2,</math> and <math>\zeta_3</math> are [[complex number]]s such that
  
<math>\begin{align*}\zeta_1+\zeta_2+\zeta_3&=1\\
+
<center><math>\begin{align*}\zeta_1+\zeta_2+\zeta_3&=1\\
 
\zeta_1^2+\zeta_2^2+\zeta_3^2&=3\\
 
\zeta_1^2+\zeta_2^2+\zeta_3^2&=3\\
\zeta_1^3+\zeta_2^3+\zeta_3^3&=7\end{align*}</math>
+
\zeta_1^3+\zeta_2^3+\zeta_3^3&=7\end{align*}</math></center>
  
 
Compute <math>\zeta_1^{7} + \zeta_2^{7} + \zeta_3^{7}</math>.
 
Compute <math>\zeta_1^{7} + \zeta_2^{7} + \zeta_3^{7}</math>.
  
 
==Solution==
 
==Solution==
{{solution}}
+
We let <math>e_1 = \zeta_1 + \zeta_2 + \zeta_3,\ e_2 = \zeta_1\zeta_2 + \zeta_2\zeta_3 + \zeta_3\zeta_1,\ e_3 = \zeta_1\zeta_2\zeta_3</math> (the [[elementary symmetric sums]]). Then, we can rewrite the above equations as
 +
<center><math>\begin{align*}\zeta_1+\zeta_2+\zeta_3&=e_1 = 1\\
 +
\zeta_1^2+\zeta_2^2+\zeta_3^2&= e_1^2 - 2e_2 = 3</math></center>
 +
from where it follows that <math>e_2 = -1</math>. The third equation can be factored as
 +
<center><math>\begin{align*}7 =\zeta_1^3+\zeta_2^3+\zeta_3^3 &= (\zeta_1+\zeta_2+\zeta_3)(\zeta_1^2+\zeta_2^2+\zeta_3^2-\zeta_1\zeta_2-\zeta_2\zeta_3 -\zeta_3\zeta_1)+3\zeta_1\zeta_2\zeta_3\\ &= e_1^3 - 3e_1e_2 + 3e_3,</math></center>
 +
from where it follows that <math>e_3 = 1</math>. Thus, applying [[Vieta's formulas]] backwards, <math>\zeta_1, \zeta_2,</math> and <math>\zeta_3</math> are the roots of the polynomial
 +
<center><math>\begin{align}x^3 - x^2 - x - 1 = 0 \Longleftrightarrow x^3 = x^2 + x + 1\end{align}</math></center>
 +
Let <math>s_n = \zeta_1^n + \zeta_2^n + \zeta_3^n</math> (the [[power sums]]). Then from <math>(1)</math>, we have the [[recursion]] <math>s_{n+3} = s_{n+2} + s_{n+1} + s_n</math>. It follows that <math>s_4 = 7 + 3 + 1 = 11, s_5 = 21, s_6 = 39, s_7 = \boxed{071}</math>.
 +
 
 +
==See also==
 +
{{Mock AIME box|year=Pre 2005|n=3|num-b=3|num-a=5}}
 +
 
 +
[[Category:Intermediate Algebra Problems]]

Revision as of 22:36, 23 April 2008

Problem

$\zeta_1, \zeta_2,$ and $\zeta_3$ are complex numbers such that

$\begin{align*}\zeta_1+\zeta_2+\zeta_3&=1\\

\zeta_1^2+\zeta_2^2+\zeta_3^2&=3\\

\zeta_1^3+\zeta_2^3+\zeta_3^3&=7\end{align*}$ (Error compiling LaTeX. Unknown error_msg)

Compute $\zeta_1^{7} + \zeta_2^{7} + \zeta_3^{7}$.

Solution

We let $e_1 = \zeta_1 + \zeta_2 + \zeta_3,\ e_2 = \zeta_1\zeta_2 + \zeta_2\zeta_3 + \zeta_3\zeta_1,\ e_3 = \zeta_1\zeta_2\zeta_3$ (the elementary symmetric sums). Then, we can rewrite the above equations as

$\begin{align*}\zeta_1+\zeta_2+\zeta_3&=e_1 = 1\\ \zeta_1^2+\zeta_2^2+\zeta_3^2&= e_1^2 - 2e_2 = 3$ (Error compiling LaTeX. Unknown error_msg)

from where it follows that $e_2 = -1$. The third equation can be factored as

$\begin{align*}7 =\zeta_1^3+\zeta_2^3+\zeta_3^3 &= (\zeta_1+\zeta_2+\zeta_3)(\zeta_1^2+\zeta_2^2+\zeta_3^2-\zeta_1\zeta_2-\zeta_2\zeta_3 -\zeta_3\zeta_1)+3\zeta_1\zeta_2\zeta_3\\ &= e_1^3 - 3e_1e_2 + 3e_3,$ (Error compiling LaTeX. Unknown error_msg)

from where it follows that $e_3 = 1$. Thus, applying Vieta's formulas backwards, $\zeta_1, \zeta_2,$ and $\zeta_3$ are the roots of the polynomial

$\begin{align}x^3 - x^2 - x - 1 = 0 \Longleftrightarrow x^3 = x^2 + x + 1\end{align}$ (Error compiling LaTeX. Unknown error_msg)

Let $s_n = \zeta_1^n + \zeta_2^n + \zeta_3^n$ (the power sums). Then from $(1)$, we have the recursion $s_{n+3} = s_{n+2} + s_{n+1} + s_n$. It follows that $s_4 = 7 + 3 + 1 = 11, s_5 = 21, s_6 = 39, s_7 = \boxed{071}$.

See also

Mock AIME 3 Pre 2005 (Problems, Source)
Preceded by
Problem 3
Followed by
Problem 5
1 2 3 4 5 6 7 8 9 10 11 12 13 14 15